Stay ahead of learning milestones! Enroll in a class over the summer!

G
Topic
First Poster
Last Poster
k a April Highlights and 2025 AoPS Online Class Information
jlacosta   0
Apr 2, 2025
Spring is in full swing and summer is right around the corner, what are your plans? At AoPS Online our schedule has new classes starting now through July, so be sure to keep your skills sharp and be prepared for the Fall school year! Check out the schedule of upcoming classes below.

WOOT early bird pricing is in effect, don’t miss out! If you took MathWOOT Level 2 last year, no worries, it is all new problems this year! Our Worldwide Online Olympiad Training program is for high school level competitors. AoPS designed these courses to help our top students get the deep focus they need to succeed in their specific competition goals. Check out the details at this link for all our WOOT programs in math, computer science, chemistry, and physics.

Looking for summer camps in math and language arts? Be sure to check out the video-based summer camps offered at the Virtual Campus that are 2- to 4-weeks in duration. There are middle and high school competition math camps as well as Math Beasts camps that review key topics coupled with fun explorations covering areas such as graph theory (Math Beasts Camp 6), cryptography (Math Beasts Camp 7-8), and topology (Math Beasts Camp 8-9)!

Be sure to mark your calendars for the following events:
[list][*]April 3rd (Webinar), 4pm PT/7:00pm ET, Learning with AoPS: Perspectives from a Parent, Math Camp Instructor, and University Professor
[*]April 8th (Math Jam), 4:30pm PT/7:30pm ET, 2025 MATHCOUNTS State Discussion
April 9th (Webinar), 4:00pm PT/7:00pm ET, Learn about Video-based Summer Camps at the Virtual Campus
[*]April 10th (Math Jam), 4:30pm PT/7:30pm ET, 2025 MathILy and MathILy-Er Math Jam: Multibackwards Numbers
[*]April 22nd (Webinar), 4:00pm PT/7:00pm ET, Competitive Programming at AoPS (USACO).[/list]
Our full course list for upcoming classes is below:
All classes run 7:30pm-8:45pm ET/4:30pm - 5:45pm PT unless otherwise noted.

Introductory: Grades 5-10

Prealgebra 1 Self-Paced

Prealgebra 1
Sunday, Apr 13 - Aug 10
Tuesday, May 13 - Aug 26
Thursday, May 29 - Sep 11
Sunday, Jun 15 - Oct 12
Monday, Jun 30 - Oct 20
Wednesday, Jul 16 - Oct 29

Prealgebra 2 Self-Paced

Prealgebra 2
Sunday, Apr 13 - Aug 10
Wednesday, May 7 - Aug 20
Monday, Jun 2 - Sep 22
Sunday, Jun 29 - Oct 26
Friday, Jul 25 - Nov 21

Introduction to Algebra A Self-Paced

Introduction to Algebra A
Monday, Apr 7 - Jul 28
Sunday, May 11 - Sep 14 (1:00 - 2:30 pm ET/10:00 - 11:30 am PT)
Wednesday, May 14 - Aug 27
Friday, May 30 - Sep 26
Monday, Jun 2 - Sep 22
Sunday, Jun 15 - Oct 12
Thursday, Jun 26 - Oct 9
Tuesday, Jul 15 - Oct 28

Introduction to Counting & Probability Self-Paced

Introduction to Counting & Probability
Wednesday, Apr 16 - Jul 2
Thursday, May 15 - Jul 31
Sunday, Jun 1 - Aug 24
Thursday, Jun 12 - Aug 28
Wednesday, Jul 9 - Sep 24
Sunday, Jul 27 - Oct 19

Introduction to Number Theory
Thursday, Apr 17 - Jul 3
Friday, May 9 - Aug 1
Wednesday, May 21 - Aug 6
Monday, Jun 9 - Aug 25
Sunday, Jun 15 - Sep 14
Tuesday, Jul 15 - Sep 30

Introduction to Algebra B Self-Paced

Introduction to Algebra B
Wednesday, Apr 16 - Jul 30
Tuesday, May 6 - Aug 19
Wednesday, Jun 4 - Sep 17
Sunday, Jun 22 - Oct 19
Friday, Jul 18 - Nov 14

Introduction to Geometry
Wednesday, Apr 23 - Oct 1
Sunday, May 11 - Nov 9
Tuesday, May 20 - Oct 28
Monday, Jun 16 - Dec 8
Friday, Jun 20 - Jan 9
Sunday, Jun 29 - Jan 11
Monday, Jul 14 - Jan 19

Intermediate: Grades 8-12

Intermediate Algebra
Monday, Apr 21 - Oct 13
Sunday, Jun 1 - Nov 23
Tuesday, Jun 10 - Nov 18
Wednesday, Jun 25 - Dec 10
Sunday, Jul 13 - Jan 18
Thursday, Jul 24 - Jan 22

Intermediate Counting & Probability
Wednesday, May 21 - Sep 17
Sunday, Jun 22 - Nov 2

Intermediate Number Theory
Friday, Apr 11 - Jun 27
Sunday, Jun 1 - Aug 24
Wednesday, Jun 18 - Sep 3

Precalculus
Wednesday, Apr 9 - Sep 3
Friday, May 16 - Oct 24
Sunday, Jun 1 - Nov 9
Monday, Jun 30 - Dec 8

Advanced: Grades 9-12

Olympiad Geometry
Tuesday, Jun 10 - Aug 26

Calculus
Tuesday, May 27 - Nov 11
Wednesday, Jun 25 - Dec 17

Group Theory
Thursday, Jun 12 - Sep 11

Contest Preparation: Grades 6-12

MATHCOUNTS/AMC 8 Basics
Wednesday, Apr 16 - Jul 2
Friday, May 23 - Aug 15
Monday, Jun 2 - Aug 18
Thursday, Jun 12 - Aug 28
Sunday, Jun 22 - Sep 21
Tues & Thurs, Jul 8 - Aug 14 (meets twice a week!)

MATHCOUNTS/AMC 8 Advanced
Friday, Apr 11 - Jun 27
Sunday, May 11 - Aug 10
Tuesday, May 27 - Aug 12
Wednesday, Jun 11 - Aug 27
Sunday, Jun 22 - Sep 21
Tues & Thurs, Jul 8 - Aug 14 (meets twice a week!)

AMC 10 Problem Series
Friday, May 9 - Aug 1
Sunday, Jun 1 - Aug 24
Thursday, Jun 12 - Aug 28
Tuesday, Jun 17 - Sep 2
Sunday, Jun 22 - Sep 21 (1:00 - 2:30 pm ET/10:00 - 11:30 am PT)
Monday, Jun 23 - Sep 15
Tues & Thurs, Jul 8 - Aug 14 (meets twice a week!)

AMC 10 Final Fives
Sunday, May 11 - Jun 8
Tuesday, May 27 - Jun 17
Monday, Jun 30 - Jul 21

AMC 12 Problem Series
Tuesday, May 27 - Aug 12
Thursday, Jun 12 - Aug 28
Sunday, Jun 22 - Sep 21
Wednesday, Aug 6 - Oct 22

AMC 12 Final Fives
Sunday, May 18 - Jun 15

F=ma Problem Series
Wednesday, Jun 11 - Aug 27

WOOT Programs
Visit the pages linked for full schedule details for each of these programs!


MathWOOT Level 1
MathWOOT Level 2
ChemWOOT
CodeWOOT
PhysicsWOOT

Programming

Introduction to Programming with Python
Thursday, May 22 - Aug 7
Sunday, Jun 15 - Sep 14 (1:00 - 2:30 pm ET/10:00 - 11:30 am PT)
Tuesday, Jun 17 - Sep 2
Monday, Jun 30 - Sep 22

Intermediate Programming with Python
Sunday, Jun 1 - Aug 24
Monday, Jun 30 - Sep 22

USACO Bronze Problem Series
Tuesday, May 13 - Jul 29
Sunday, Jun 22 - Sep 1

Physics

Introduction to Physics
Wednesday, May 21 - Aug 6
Sunday, Jun 15 - Sep 14
Monday, Jun 23 - Sep 15

Physics 1: Mechanics
Thursday, May 22 - Oct 30
Monday, Jun 23 - Dec 15

Relativity
Sat & Sun, Apr 26 - Apr 27 (4:00 - 7:00 pm ET/1:00 - 4:00pm PT)
Mon, Tue, Wed & Thurs, Jun 23 - Jun 26 (meets every day of the week!)
0 replies
jlacosta
Apr 2, 2025
0 replies
Fibonacci sequence
April   1
N 15 minutes ago by ririgggg
Source: Vietnam NMO 1989 Problem 2
The Fibonacci sequence is defined by $ F_1 = F_2 = 1$ and $ F_{n+1} = F_n +F_{n-1}$ for $ n > 1$. Let $ f(x) = 1985x^2 + 1956x + 1960$. Prove that there exist infinitely many natural numbers $ n$ for which $ f(F_n)$ is divisible by $ 1989$. Does there exist $ n$ for which $ f(F_n) + 2$ is divisible by $ 1989$?
1 reply
April
Feb 1, 2009
ririgggg
15 minutes ago
Prove that x1=x2=....=x2025
Rohit-2006   4
N 19 minutes ago by kamatadu
Source: A mock
The real numbers $x_1,x_2,\cdots,x_{2025}$ satisfy,
$$x_1+x_2=2\bar{x_1}, x_2+x_3=2\bar{x_2},\cdots, x_{2025}+x_1=2\bar{x_{2025}}$$Where {$\bar{x_1},\cdots,\bar{x_{2025}}$} is a permutation of $x_1,x_2,\cdots,x_{2025}$. Prove that $x_1=x_2=\cdots=x_{2025}$
4 replies
Rohit-2006
Today at 5:22 AM
kamatadu
19 minutes ago
Locus of a point on the side of a square
EmersonSoriano   1
N 37 minutes ago by vanstraelen
Source: 2018 Peru Southern Cone TST P7
Let $ABCD$ be a fixed square and $K$ a variable point on segment $AD$. The square $KLMN$ is constructed such that $B$ is on segment $LM$ and $C$ is on segment $MN$. Let $T$ be the intersection point of lines $LA$ and $ND$. Find the locus of $T$ as $K$ varies along segment $AD$.
1 reply
EmersonSoriano
Apr 2, 2025
vanstraelen
37 minutes ago
NT function debut
AshAuktober   2
N 38 minutes ago by Kazuhiko
Source: 2025 Nepal Practice TST 3 P2 of 3; Own
Let $f$ be a function taking in positive integers and outputting nonnegative integers, defined as follows:
$f(m)$ is the number of positive integers $n$ with $n \le m$ such that the equation $$an + bm = m^2 + n^2 + 1$$has an integer solution $(a, b)$.
Find all positive integers $x$ such that$f(x) \ne 0$ and $$f(f(x)) = f(x) - 1.$$Adit Aggarwal, India.
2 replies
AshAuktober
2 hours ago
Kazuhiko
38 minutes ago
No more topics!
Showing properties about subsets of positive integers.
Functional   41
N Apr 1, 2025 by maromex
Source: IMO 2018 Shortlist A3
Given any set $S$ of positive integers, show that at least one of the following two assertions holds:

(1) There exist distinct finite subsets $F$ and $G$ of $S$ such that $\sum_{x\in F}1/x=\sum_{x\in G}1/x$;

(2) There exists a positive rational number $r<1$ such that $\sum_{x\in F}1/x\neq r$ for all finite subsets $F$ of $S$.
41 replies
Functional
Jul 17, 2019
maromex
Apr 1, 2025
Showing properties about subsets of positive integers.
G H J
G H BBookmark kLocked kLocked NReply
Source: IMO 2018 Shortlist A3
The post below has been deleted. Click to close.
This post has been deleted. Click here to see post.
Functional
18 posts
#1 • 4 Y
Y by centslordm, ImSh95, GeoMetrix, Adventure10
Given any set $S$ of positive integers, show that at least one of the following two assertions holds:

(1) There exist distinct finite subsets $F$ and $G$ of $S$ such that $\sum_{x\in F}1/x=\sum_{x\in G}1/x$;

(2) There exists a positive rational number $r<1$ such that $\sum_{x\in F}1/x\neq r$ for all finite subsets $F$ of $S$.
This post has been edited 2 times. Last edited by v_Enhance, Jul 26, 2020, 7:14 PM
Reason: spelling
Z K Y
The post below has been deleted. Click to close.
This post has been deleted. Click here to see post.
Kayak
1298 posts
#2 • 2 Y
Y by ImSh95, Adventure10
Also Indian TST 3 P1
Z K Y
The post below has been deleted. Click to close.
This post has been deleted. Click here to see post.
rkm0959
1721 posts
#3 • 7 Y
Y by Starrysky, Smkh, centslordm, bigboo, ImSh95, Adventure10, Mango247
Fun :)
If $S$ is finite, it is clear that (2) holds, as there are infinitely many rational numbers less than 1.
Therefore, let us assume that $S$ is infinite. We will prove this problem indirectly. Assume that (1), (2) both fail.
Now we get that there exists a unique finite subset $S(r) \subset S$ such that $\sum_{x \in S(r)} \frac{1}{x} = r$ for each $r<1$.
We will show that this is impossible, which shows the desired contradiction. We can also now easily assume that $1 \notin S$, since if there is a set $S$ which satisfies the condition and also has $1 \in S$, then $S - \{1\}$ satisfies the condition as well.
With this big condition, we can force a lot of good conditions on $S(r)$.
For example, if we have an element $x \in S - S(r)$, we can immediately force $S \left( r + \frac{1}{x} \right) = S(r) \cup \{x\}$, if $r+ \frac{1}{x}<1$.

Let us label the elements of $S$ as $2 \le u_1 < u_2 < \cdots $.
If there exists an $n$ such that $u_{n+1} < 2u_n$, let us denote $r = \frac{1}{u_n} - \frac{1}{u_{n+1}} < \frac{1}{u_{n+1}}$
Because $r<1$, there should be a unique set $S(r)$ which satisfies $\sum_{x \in S(r)} \frac{1}{x} = r$.
Clearly, for any integer $x$ in $S(r)$, we will have $x > u_{n+1}$. Therefore, $u_{n+1}$ is definitely not in $S(r)$.
Now, it is clear that $T = S(r) \cup \{u_{n+1}\}$ satisfies $\sum_{x \in T} \frac{1}{x} = \frac{1}{u_n}$.
However, $T' = \{u_n \}$ also satisfies $\sum_{x \in T'} \frac{1}{x} = \frac{1}{u_n}$, so this shows that (1) is true. Contradiction.

Now we have $u_{n+1} \ge 2u_n$ for all $n$. In this case, we can see that $\sum_{i=1}^\infty \frac{1}{u_i}$ can't be too big.
Indeed, the largest possible value of $\sum_{i=1}^\infty \frac{1}{u_i}$ is $\sum_{i=1}^\infty \frac{1}{2^i} = 1$.
Therefore, if $u_i \neq 2^i$ for some $i$, we immediately have $\sum_{i=1}^\infty \frac{1}{u_i} < 1$.
Taking $Q = \sum_{i=1}^\infty \frac{1}{u_i} < 1$, we can just take $r = \frac{1+Q}{2}$ and see that (2) is true. Contradiction.

Now we have $u_n = 2^n$ for all $n$. This is quite easy to do.
For all finite subsets $T$ of $S$, the denominator of $\sum_{x \in T} \frac{1}{x}$ will be a power of 2.
So we can just take something like $r = \frac{2018}{2019}$ and call it a day. Contradiction.
This post has been edited 2 times. Last edited by rkm0959, Jul 17, 2019, 1:10 PM
Z K Y
The post below has been deleted. Click to close.
This post has been deleted. Click here to see post.
MarkBcc168
1594 posts
#4 • 14 Y
Y by kevinatcausa, XbenX, mijail, rashah76, qubatae, Siddharth03, rcorreaa, snap7822, ImSh95, Quidditch, sabkx, Adventure10, Mango247, Sedro
Assume that both assertions are false. Clearly $S$ is infinite. We first extract the uniqueness condition.

Claim: There does not exists $a,b\in S$ such that $a<b<2a$.

Proof: Take subset $F$ such that $\sum\limits_{x\in F}\frac{1}{x} = \frac{1}{b}-\frac{1}{a}$. Clearly $a\notin F$ by size reasons. So notice that
$$\sum_{x\in F\cup\{a\}}\frac{1}{x} = \frac{1}{a}+\sum\limits_{x\in F}\frac{1}{x}=\frac{1}{b},$$contradiction to the first assertion.
Enumerate $S=\{a_1,a_2,a_3,...\}$ where $a_1<a_2<...$. By the first claim $a_{i+1} > 2a_i$. Now we extract the existence condition.

Claim: $a_{i+1} = 2a_i$ for any $i\in\mathbb{Z}^+$.

Proof: Assume that $a_{i+1} > 2a_i$. Take any rational $r\in\left(\frac{2}{a_{i+1}},\frac{1}{a_i}\right)$ and set $F\subset S$ such that $\sum\limits_{x\in F}\frac{1}{x} = r$. Clearly $\{a_1,a_2,...,a_i\}\notin F$ for size reason. Thus
$$r < \frac{1}{a_{i+1}} + \frac{1}{a_{i+2}} + \frac{1}{a_{i+3}} + ... \leqslant \frac{1}{a_{i+1}}\left(1+\frac{1}{2}+\frac{1}{4}+\frac{1}{8}+...\right)=\frac{2}{a_{i+1}},$$contradiction.
Thus set $S$ must be in form $\{k, 2k, 4k, 8k,...\}$. However, this makes $\frac{1}{3k}$ inexpressible, final contradiction.
Z K Y
The post below has been deleted. Click to close.
This post has been deleted. Click here to see post.
plagueis
157 posts
#5 • 3 Y
Y by ImSh95, Adventure10, Mango247
Loved this problem.
Z K Y
The post below has been deleted. Click to close.
This post has been deleted. Click here to see post.
TheDarkPrince
3042 posts
#6 • 2 Y
Y by ImSh95, Adventure10
Functional wrote:
Given any set $S$ of postive integers, show that at least one of the following two assertions holds:

(1) There exist distinct finite subsets $F$ and $G$ of $S$ such that $\sum_{x\in F}1/x=\sum_{x\in G}1/x$;

(2) There exists a positive rational number $r<1$ such that $\sum_{x\in F}1/x\neq r$ for all finite subsets $F$ of $S$.

Solution. Assume that both the assertions do not hold. As second condition does not hold, $S$ has the infinite. Therefore for any rational $r<1$, there is an unique subset of $S$, $F$ such that $\sum_{x\in F} \frac{1}{x} = r$.

Suppose there are elements $a,b$ in $S$ such that $a<b<2a$. Then there is a subset $F$ such that $\sum_{x\in F} \frac{1}{x} = \frac{1}{a} - \frac{1}{b}$. If $b\in F$, then $\frac{1}{a} = \sum_{x\in F} \frac{1}{x} + \frac{1}{b} \geq \frac{2}{b}$ which is a contradiction as we have two subsets with same sum.

Suppose $S = \{a_1,a_2,\ldots \}$, and $a_i$'s are in increasing order. We have $a_{n+1} > 2a_n$ for all $n$. Therefore $a_n > 2^{n-1}a_1$. Therefore \[\sum_{i=1} \frac{1}{a_i} < \frac{1}{a_1} \left(1+\frac{1}{2} + \frac{1}{4} + \ldots \right)<\frac{2}{a_1}.\]If $a_1 \geq 2$, $\sum \frac{1}{a_1}$ is strictly less than $1$, and we can pick a rational between $1$ and that number which gives a contradiction as second condition was false. Therefore $a_1 = 1$. Now for a rational $r<1$, we have a subset $F$ such that $\sum _{x\in F}\frac{1}{x} = r<1$. Therefore $a_1\not\in F$ and again we get $\sum_{i=2} \frac{1}{a_i} < 1$ which again gives a contradiction to the second condition and we are done. $~\square$
Z K Y
The post below has been deleted. Click to close.
This post has been deleted. Click here to see post.
prague123
230 posts
#7 • 3 Y
Y by ImSh95, Adventure10, Mango247
plagueis wrote:
Loved this problem.

It is not a good IMO problem as it does not clearly fit into the ACGN scheme. It is not A and also not C.
Z K Y
The post below has been deleted. Click to close.
This post has been deleted. Click here to see post.
Axiomatical
32 posts
#8 • 4 Y
Y by DVDthe1st, ImSh95, Adventure10, Mango247
prague123 wrote:
plagueis wrote:
Loved this problem.

It is not a good IMO problem as it does not clearly fit into the ACGN scheme. It is not A and also not C.

There are examples of such problems on the IMO in several places, just look at IMO 2014/5. A problem being between categories does not disqualify it, and this is a very nice problem.

Besides, it is much, much more A than C, merely nonstandard A in the style of IMOSL 2016 A2, A3 or 2017 A3.
Z K Y
The post below has been deleted. Click to close.
This post has been deleted. Click here to see post.
yayups
1614 posts
#9 • 3 Y
Y by ImSh95, SerdarBozdag, Adventure10
Assume for sake of contradiction that neither of the statements is true. Then, for every rational number $r\in(0,1)$, we have a unique finite set $F(r)\subset S$ such that
\[r=\sum_{x\in F(r)}1/x.\]For the purposes of this assertion, having $1$ in $S$ won't affect anything since $1\not\in F(r)$ for all $r\in(0,1)$ as $r<1$, so WLOG assume $1\not\in S$.

Suppose $x,y\in S$ with $x>y$. Then
\[1/x+(1/y-1/x)=1/y,\]so if $x\not\in F(1/y-1/x)$ then we have two different ways of representing $1/y$ namely $\{y\}$ and $F(1/y-1/x)\cup\{x\}$. Thus, $x\in F(1/y-1/x)$, so
\[1/x\le 1/y-1/x\]which means that $x\ge 2y$. So if we order $S$ as
\[S=\{x_1<x_2<\cdots\},\]we must have $x_{k+1}\ge 2x_k$. This means that $x_{k+1}\ge 2^k x_1$.

However, note that if $r$ can be made, then certainly
\[r<\sum_{x\in S}1/x.\]This is true for $r$ arbitrarily close to $1$, so
\[\sum_{x\in S}1/x\ge 1.\]But since $x_{k+1}\ge 2^k x_1$, $\sum_{x\in S}1/x\le 2/x_1$, so $2/x_1\ge 1$, so $x_1=2$. Thus,
\[S=\{2,4,8,16,32,\ldots\}.\]Clearly $1/3$ is not a finite sum of these, so we have our desired contradiction.
Z K Y
The post below has been deleted. Click to close.
This post has been deleted. Click here to see post.
yunxiao
21 posts
#10 • 5 Y
Y by ImSh95, Adventure10, Mango247, Mango247, Mango247
I think it is trivial,because card(2^S)>card(Q),isn’t it correct?
Z K Y
The post below has been deleted. Click to close.
This post has been deleted. Click here to see post.
yayups
1614 posts
#11 • 2 Y
Y by ImSh95, Adventure10
@above
Finite subsets of S. Cardinality of those is same as Q
Z K Y
The post below has been deleted. Click to close.
This post has been deleted. Click here to see post.
62861
3564 posts
#12 • 3 Y
Y by aopsuser305, ImSh95, Adventure10
Compare with https://artofproblemsolving.com/community/c6h37239p233042
Z K Y
The post below has been deleted. Click to close.
This post has been deleted. Click here to see post.
yayups
1614 posts
#13 • 2 Y
Y by ImSh95, Adventure10
@above
huh this is basically the exact same problem....
Z K Y
The post below has been deleted. Click to close.
This post has been deleted. Click here to see post.
62861
3564 posts
#14 • 4 Y
Y by yayups, aopsuser305, ImSh95, Adventure10
Now you know why this didn't appear on a mop test
Z K Y
The post below has been deleted. Click to close.
This post has been deleted. Click here to see post.
william122
1576 posts
#15 • 2 Y
Y by ImSh95, Adventure10
Suppose the contrary, namely that all rationals $0<r<1$ can be expressed uniquely. Clearly, this means that $S$ is infinite. Also, suppose $1\not\in S$, since this relaxes condition 1, and doesn't impact 2. Finally, define $S_r$ to be the unique subset of $S$ which generates $r$.

Given $n\in \mathbb{Z}^+$, suppose that there exists $x\in S\cap[n,2n)$ such that $x\not\in S_{1/n}$. Then, the set $S_{1/n-1/x}\cup\left\{x\right\}$ also generates $1/n$, which contradicts uniqueness. Therefore, such an $x$ doesn't exist. So, all $S\cap[n,2n)\subseteq S_{1/n}$. However, the sum of reciprocals of 2 numbers in this range already exceeds $1/n$, so $|S\cap[n,2n)|\le 1$. Now, consider the intervals $[2,4)$, $[4,8)$, etc. As there is at most 1 number from $S$ in each set, $\sum\limits_{x\in S}1/x\le 1$, with equality iff $S=\left\{2^x\arrowvert x\ge 1\right\}$. Unfortunately, this choice of $S$ doesn't work, so the inequality is strict, and we can find a sufficiently small epsilon such that $1-\epsilon$ is unexpressible. This is a contradiction, as desired.
This post has been edited 2 times. Last edited by william122, Aug 7, 2019, 4:54 PM
Z K Y
The post below has been deleted. Click to close.
This post has been deleted. Click here to see post.
IndoMathXdZ
691 posts
#16 • 2 Y
Y by ImSh95, Adventure10
Assume that both assertion doesn't hold, that is, there exists a set $S$ of positive integers such that
For every positive rational number $r < 1$, there exists a unique finite subset $F$ of $S$ such that
\[ \sum_{x \in F} \frac{1}{x} = r \]
Obviously $S$ is infinite, otherwise take $r < \frac{1}{\max S}$ and this contradicts the second statement.

Claim 01. There doesn't exists any two element $a,b \in S$ such that $a < b < 2a$.
Proof. Suppose otherwise, then we have $\frac{1}{b} < \frac{1}{a} < \frac{2}{b}$. Therefore, we can construct $\frac{1}{a}$ in two ways: which is just the partition of $\frac{1}{a}$, which exists by the hypothesis, or $\frac{1}{b}$ and the partition of $\left( \frac{1}{a} - \frac{1}{b} \right)$, a contradiction.

Now, consider the elements of $S$ ordered from the smallest number: $a_1 < a_2 < a_3 < \dots$
By Claim 01, we have $a_{i + 1} \ge 2a_i$ for every $i \in \mathbb{N}$.
We could WLOG $a_1 \not= 1$, as whether $1$ is in or not won't affect the problem as the hypothesis only holds for $r < 1$.
Claim 02. $a_1 = 2$.
Proof. Suppose otherwise, that $a_1 = 3$. Then
\[ \sum_{x \in S} \frac{1}{x} \le \sum_{i = 0}^{\infty} \frac{1}{2^i \cdot 3} = \frac{2}{3} \]Taking $r > \frac{2}{3}$, this clearly won't satisfy.
In a similar method, one can prove that $a_i = 2^i$ for every $i \in \mathbb{N}$.
Now, we'll prove that $r = \frac{1}{3}$ is not attainable.
To prove this, we claim that the denominator of $r$ will stay $0$ modulo $3$ at the whole process of subtraction.
\[ r - \frac{1}{2^k} = \frac{1}{3 \cdot N} - \frac{1}{2^k} = \frac{2^k - 3 \cdot N}{3 \cdot N \cdot 2^k} \]It's clear enough that $2^k - 3 \cdot N$ is not $0$ modulo $3$, and therefore we've finished.
Z K Y
The post below has been deleted. Click to close.
This post has been deleted. Click here to see post.
Plops
946 posts
#17 • 1 Y
Y by ImSh95
In my proof, I implicitly use the fact that $\sum_{n=1}^{\infty} \frac{1}{n}$ diverges.

Assume $(2)$ doesn't hold. Then, clearly $S$ is an infinite set of positive integers. Given two finite subsets $A$ and $B$ of $S$, and WLOG $\sum_{x \in A}^{} \frac{1}{x} > \sum_{x \in B}^{} \frac{1}{x}$. It suffices to prove
$$\sum_{x \in A}^{}\frac{1}{x}=r \text{ and } \sum_{x \in B}^{}\frac{1}{x}=r-\epsilon \text{ such that }\epsilon<\text{ min }{\frac{1}{x} : x \in A \cup B}$$Consider successively deleting $\frac{1}{l}$ from $\sum_{x \in A}^{}\frac{1}{x}$ where $l$ is the maximal element of $A$, and let $A$ now be the new sequence of integers obtained from deleting $l$. We do this until $\sum_{x \in A \setminus{j}}^{} \frac{1}{x} < \sum_{x \in B}^{} \frac{1}{x}$ where $j$ is the maximum element set $A$.
We consider our current sets $A$ and $B$ from now on. Like in the above notation, let $\sum_{x \in A}^{} \frac{1}{x}-\sum_{x \in B}^{} \frac{1}{x}=\epsilon$. Furthermore, let $j$ be the maximal element of $A$. Then, $\sum_{x \in A \setminus{j}}^{} \frac{1}{x} < \sum_{x \in B}^{} \frac{1}{x} \implies \epsilon=\sum_{x \in A}^{} \frac{1}{x}-\sum_{x \in B}^{} \frac{1}{x}< \frac{1}{j}$.
Then, let $w$ be the maximal element of $B$ and consider the set $B'=\{u \in \mathbb{Z}: u>w\}$. Now, consider adding the least remaining element of $B'$, $m$ for convenience to $B$, and call the new set $B \cup \{m\}$ as $B$, and let $\sum_{x\in A}^{}\frac{1}{x}-\sum_{x\in B}^{}\frac{1}{x}=\epsilon$ as before. We keep applying the above operation to $G$ until $\epsilon>0$ and applying the operation $1$ more time results in $\epsilon<0$. Let the number we add to $G$ such that $\epsilon<0$ be $l$. Then
$$\sum_{x \in A}^{} \frac{1}{x}-\sum_{x\in B}^{} \frac{1}{x}-\frac{1}{l}<0 \implies \epsilon-\frac{1}{l}<0 \implies \epsilon<\frac{1}{l}$$However, $\epsilon<\{\frac{1}{x}:x \in A \} \text{ and } \epsilon<\frac{1}{l}<\{\frac{1}{x}: x \in B \}$.
Therefore, given $(2)$ does not hold, we can find finite subsets $A$ and $B$ of $S$ such that $\sum_{x \in A}^{}\frac{1}{x}=r \text{ and } \sum_{x \in B}^{}\frac{1}{x}=r-\epsilon \text{ such that }\epsilon<\{\text{ min }{\frac{1}{x} : x \in A \cup B}\}$, and there must exist a finite subset $L$ of $S$ completely distinct from $F$ and $G$ such that $$\sum_{x\in L}^{} \frac{1}{x}=\epsilon \implies \sum_{x \in A}^{} \frac{1}{x}=\sum_{x \in B \cup L}^{} \frac{1}{x}$$
Otherwise, $(2)$ holds, and we are done.
This post has been edited 3 times. Last edited by Plops, Mar 25, 2020, 3:08 PM
Z K Y
The post below has been deleted. Click to close.
This post has been deleted. Click here to see post.
TheUltimate123
1740 posts
#18 • 2 Y
Y by ImSh95, sabkx
Solved with eisirrational and Th3Numb3rThr33.

Assume for contradiction both fail. Then for each $r<1$ there is a unique finite subset $R(r)$ of $S$ such that $\textstyle\sum_{x\in F}1/x=r$.

Claim: There are no two elements $a,b\in S$ with $a<b<2a$.

Proof. Let $a,b\in S$ with $a<b$, so that $R(1/a)=1/a$ and $R(1/b)=1/b$. Note that if \[\frac1b\notin R\left(\frac1a-\frac1b\right),\]then $R(1/a-1/b)\cup\{1/b\}$ is another way to represent $1/a$, contradiction. Hence \[\frac1a-\frac1b\ge\frac1b\implies b\ge2a,\]as claimed. $\blacksquare$


Now if $S$ is finite, then (ii) clearly holds, and if $1\in S$, we may delete it since it is not $R(r)$ for all $r<1$. Thus we have \[\sum_{x\in S}\frac1x\le\frac12+\frac14+\frac18+\cdots=1,\]where equality holds if and only if $S=\{2,4,8,\ldots\}$. If equality fails, then some $r<1$ cannot be represented, but if $S=\{2,4,8,\ldots\}$, then it is impossible to represent $1/3$, the end.
Z K Y
The post below has been deleted. Click to close.
This post has been deleted. Click here to see post.
Remon
20 posts
#19 • 1 Y
Y by ImSh95
There are many similar question.
Z K Y
The post below has been deleted. Click to close.
This post has been deleted. Click here to see post.
Plops
946 posts
#20 • 1 Y
Y by ImSh95
Indeed: Problem $6$ from this has a similar taste.
This post has been edited 1 time. Last edited by Plops, Mar 30, 2020, 2:56 PM
Z K Y
The post below has been deleted. Click to close.
This post has been deleted. Click here to see post.
hansu
300 posts
#21 • 1 Y
Y by ImSh95
Functional wrote:
Given any set $S$ of postive integers, show that at least one of the following two assertions holds:

(1) There exist distinct finite subsets $F$ and $G$ of $S$ such that $\sum_{x\in F}1/x=\sum_{x\in G}1/x$;

(2) There exists a positive rational number $r<1$ such that $\sum_{x\in F}1/x\neq r$ for all finite subsets $F$ of $S$.

Assume to the contrary such a set exists. We begin by observing that $S$ must be infinite. Now suppose that $S=\{a_1,a_2,\ldots\}$, where $a_{i+1}>a_i$. Also observe that $1\not\in S$. If we had $\frac{1}{a_i}-\frac{1}{a_{i+1}}< \frac{1}{a_{i+1}}$, then we are violating (2). Then we must have $\frac{1}{a_i}\ge \frac{2}{a_{i+1}}\implies a_{i+1}\ge 2a_i$. Thus we must have $a_i\ge 2^i$. Suppose $N$ is the smallest positive integer such that $a_N>2^N$. Then we have
$$\sum_{x\in S}<1$$This again violates (2). But if $S=\{2,4,8,\ldots\}$, we can not represent $\frac{1}{3}$ using finitely many elements of $S$, a contradiction.
Z K Y
The post below has been deleted. Click to close.
This post has been deleted. Click here to see post.
niyu
830 posts
#22 • 1 Y
Y by ImSh95
Really nice problem!

If $S$ is a finite set, there are only a finite number of subsets of $S$; hence, as there are an infinite number of rational numbers $r < 1$, condition (2) is true. Henceforth, we assume $S$ is an infinite set.

Suppose condition (2) is false, i.e. for each rational number $r < 1$, there exists a finite subset $F_r$ of $S$ such that $\sum_{x \in F_r} \frac{1}{x} = r$. In what follows, we assume that $1$ is not an element of $S$; since $1 \not\in F_r$, removing $1$ from $S$ if it exists will not alter the truth of condition (2). We will show that condition (1) is true.

Main Claim: There exist $u, v \in S$ for which $u > v > \frac{u}{2}$.

Proof: Suppose not, and let the elements of $S$ be $1 < a_1 < a_2 < \cdots$. For each $i > 1$, we have $\frac{a_{i + 1}}{a_i} \geq 2$, so $a_k \geq 2^{k - 1}a_1$. Hence, we have
\begin{align*}
        T &= \sum_{i = 1}^\infty \frac{1}{a_i} \\
        &\leq \sum_{i = 1}^{\infty} \frac{1}{2^{i - 1}a_1} \\
        &\leq \frac{1}{a_1}\sum_{i = 0}^\infty \frac{1}{2^i} \\
        &\leq \frac{2}{a_1} \\
        &\leq 1.
\end{align*}Equality holds iff $a_k = 2^k$ for each $k$. If $T < 1$, pick a rational number $r$ for which $1 > r > T$. We have
\begin{align*}
        r = \sum_{x \in F_r} \frac{1}{x} < \sum_{x \in S} \frac{1}{x} = T < r,
\end{align*}contradiction. Hence, we must have $T = 1$, implying that $a_k = 2^k$ for each $k$. However, as $S$ is the set of powers of $2$ larger than $1$, there is no finite subset $F \in S$ such that $\sum_{x \in F} \frac{1}{x} = 0.\overline{1}_2 = \frac{1}{3}$, contradicting the existence of $F_{\frac{1}{3}}$. Thus, we have reached a contradiction, proving the claim. $\blacksquare$

Now, consider $u, v \in S$ for which $u > v > \frac{u}{2}$. Note that $\frac{1}{v} - \frac{1}{u} < \frac{1}{u}$, so $F_{\frac{1}{v} - \frac{1}{u}}$ does not contain $u$. Let $F = F_{\frac{1}{v} - \frac{1}{u}} \cup \{u\}$. We have $F \neq \{v\}$, and $\sum_{x \in F} \frac{1}{x} = \sum_{x \in \{v\}} \frac{1}{x}$. Thus, condition (1) is true, so we are done. $\Box$
This post has been edited 2 times. Last edited by niyu, Jul 23, 2020, 12:52 AM
Z K Y
The post below has been deleted. Click to close.
This post has been deleted. Click here to see post.
blacksheep2003
1081 posts
#23 • 1 Y
Y by ImSh95
Solution
This post has been edited 2 times. Last edited by blacksheep2003, Oct 18, 2020, 3:46 AM
Z K Y
The post below has been deleted. Click to close.
This post has been deleted. Click here to see post.
jj_ca888
2726 posts
#24 • 1 Y
Y by ImSh95
FTSoC assume both are false. Let $s(\mathcal{F})$ for every $\mathcal{F} \subseteq S$ be the sum of $\tfrac 1x$ over all $x \in \mathcal{F}$. Then we have $s$ is injective, and surjective on $(0, 1)$. We claim, that we can actually uniquely define $S$.

Let $x, y$ be elements of $S$. WLOG $1 \not \in S$ since we are only given the surjective condition on $< 1$, so this $1$ term would not matter. WLOG $x > y$, then if $\tfrac 1y - \tfrac 1x \in (0, 1)$ note that $x$ must be in the set $F$ for which $s(F) = \tfrac 1y - \tfrac 1x$, else there are two ways of representing $s(F)$, by simply using $y$ or using both $F$ combined with $x$. So this tells is $x \geq 2y$ by bounding.

Next, we rank $S$'s elements in order $a_1 < a_2 < \ldots$ where by the previously derived condition, clearly $a_{i+1} \geq 2a_i$. We will show that in fact, equality must hold. If not, then choosing $r = 1 - \epsilon$ cannot be achieved. Hence indeed, we can directly classify $S$ as the set of all powers of $2$ at least $2$. But this set clearly is not surjective, contradiction. $\blacksquare$
Z K Y
The post below has been deleted. Click to close.
This post has been deleted. Click here to see post.
pipitchaya.s4869
18 posts
#25 • 1 Y
Y by ImSh95
Also Thailand TST 2019
Z K Y
The post below has been deleted. Click to close.
This post has been deleted. Click here to see post.
Diego17
29 posts
#26 • 1 Y
Y by ImSh95
assume that both statement fail
For each r there exists F such that the sum of the reciprocal of the numbers in F is r . Suppose F={a1,a2,……aj}
obviously, we have S is infinite, othwise statement 2 holds.
(1) if there exists an number m such that r/2<m≤r and m is not in F
use statement 2, we have a subset F1 such that the sum of the reciprocal of the numbers in F is r-m
notice that r-m < r/2 < m so m is not in F1
regard F1∪{m} and F, we have a contradiction.
(2) else, suppose a is in S, regard the region (a,2a)
let r be 2a by using (1), we have that there's no number in that region which is also in S
using the same method, we have: The maximum of the sum of the reciprocal of the numbers in S and ≤a is a+2/a+4/a+……
let r be 2a-ε so 2/a,4/a,…… is all the number in S and <a but since all numbers in S are intigers, we have a contradiction.
Z K Y
The post below has been deleted. Click to close.
This post has been deleted. Click here to see post.
peatlo17
77 posts
#27 • 1 Y
Y by ImSh95
Let $A$ be the set $\{1/x \mid x \in S\}$. Clearly, $A$ contains an infinite number of elements. Let the elements of $A$ be $a_1 \ge a_2 \ge a_3 \ge \dots$
For some $r \in \mathbb{Q}$, let $A(r)$ denote the set of elements in $A$ which sum to $r$.

FTSOC, assume that both conditions are false.

Then, from condition (1) we have that all elements of $A$ are distinct. Furthermore, from (2) we have the following:

Claim: $a_{i+1} \le a_{i}/2$
Proof. Otherwise, we can choose $r$ such that $2a_i > 2a_{i+1} > r > a_i$. Then, $r-a_i < a_i$ and $r-a_{i+1} < a_{i+1}$ so the distinct sets $a_i \cup A(r-a_i)$ and $a_{i+1} \cup A(r-a_{i+1})$ both sum to $r$, contradiction. $\square$

As we are considering $r<1$, we can assume that $a_1 \le 1/2$. Then the previous claim implies that $a_2 \le 1/4$, $a_3 \le 1/8$, and so on. If one of the preceding inequalities is strict, then the sum of all elements of $A$ is less than 1, contradicting (2). Otherwise, the elements of $A$ must be $1/2, 1/4, 1/8, 1/16, \dots$, so numbers like $1/7$ cannot be written as a sum of distinct elements of $A$, contradiction.

Remark
This post has been edited 1 time. Last edited by peatlo17, Jun 13, 2021, 2:29 PM
Z K Y
The post below has been deleted. Click to close.
This post has been deleted. Click here to see post.
cmsgr8er
434 posts
#28 • 1 Y
Y by ImSh95
Suppose both statements' negations are true. In other words, every rational number $r<1$ can be expressed as $\sum_{x\in F} 1/x = r$ for some finite subset $F$ and such subset is unique.

Claim: There cannot exist $a,b\in S$ with $a<b < 2a$.

Proof. By assumption there then exists subset $F$ with
$$\sum_{x\in F} 1/x = \frac1a - \frac1b < \frac 2b - \frac 1b = \frac 1b < \frac{1}{a},$$so $a,b\not\in F,$ implying that
$$\sum_{x\in F\cup\{b\}} = \frac{1}{a},$$Contradiction. $\blacksquare$

Claim: For all $x>1$ with $x\in S,$ $\sum_{x\in S} 1/x \ge 1.$

Proof. Suppose not, and that $\sum_{1<x \in S} 1/x = r < 1,$ then $r+\varepsilon < 1$ for sufficiently small $\varepsilon \in \mathbb Q$ cannot be obtained, contradiction. $\blacksquare$

However, it is obvious then from our first claim that,
$$\sum_{1< x \in S} 1/x \le \sum_{i=1}^\infty \frac{1}{2^i} = 1,$$So from our earlier claim, this forces $S = \{2,4,8,\cdots\}.$ Consequently, each rational $r<1$ is uniquely expressible as its binary representation. However, $2/3 = 0.\overline{10}_2$ is infinitely periodic, meaning no such finite subset of $S$ has elements' sum of $2/3,$ contradiction. Therefore, we conclude at least one of the statements must be true. $\blacksquare$
Z K Y
The post below has been deleted. Click to close.
This post has been deleted. Click here to see post.
508669
1040 posts
#29 • 1 Y
Y by ImSh95
Functional wrote:
Given any set $S$ of positive integers, show that at least one of the following two assertions holds:

(1) There exist distinct finite subsets $F$ and $G$ of $S$ such that $\sum_{x\in F}1/x=\sum_{x\in G}1/x$;

(2) There exists a positive rational number $r<1$ such that $\sum_{x\in F}1/x\neq r$ for all finite subsets $F$ of $S$.

FTSoC such a set $S$ exists which satisfied both the conditions. The second condition implies that $|S|$ is infinite. If there exists $x, y \in S$ such that $y \in [x, 2x]$, then we see that choosing elements $s_1, s_2 \dots s_k \in S$ such that $\sum\limits_{i=1}^k s_i = \frac{1}{x} - \frac{1}{y} < \frac{1}{y}$, then $\sum\limits_{i=1}^k s_i + \frac{1}{y} = \frac{1}{x}$, a contradiction to condition $2$. We now remove $1$ from $S$ as it is not useful to $S$ now. We see that $\sum\limits_{x \in S} \frac{1}{x} \leq \sum\limits_{i=1}^{\infty} \frac{1}{2^i} = 1$, which means that if $S \neq \{ 2, 4, 8 \dots \}$ then there exists arbitrarily small positive reals $\epsilon$ for which $1 - \epsilon$ cannot be represented as $\sum\limits_{i=1,x_i\in S}^k \frac{1}{x_i} = 1-\epsilon$ and if $S = \{2, 4, 8 \dots \}$ then any fraction $\frac{p}{q}$ with $\text{rad}(q) \neq 2^k$ for some $k \in \mathbb{Z}^+$ cannot be represented as in the second condition, a contradiction. Therefore no such set $S$ which satisfies both the conditions.
This post has been edited 1 time. Last edited by 508669, Aug 25, 2021, 11:16 AM
Z K Y
The post below has been deleted. Click to close.
This post has been deleted. Click here to see post.
rcorreaa
238 posts
#30 • 1 Y
Y by ImSh95
Assume that both conditions are not satisfied.

Let $s$ be the smallest element of $S$. Suppose that there is an $r \in S$ such that $s<r<2s$. Now, observe that there exists a subset $F$ of $S$ such that $\frac{1}{s}-\frac{1}{r}= \sum_{x \in F} \frac{1}{x}$. Observe that if $r \not \in F \implies \frac{1}{s}= \sum_{x \in F \cup \{r\}} \frac{1}{x}$, a contradiction since $s \in S$. Thus, $r \in F \implies \frac{1}{2s} > \frac{1}{s}-\frac{1}{r} \geq \frac{1}{r}> \frac{1}{2s}$, contradiction.

Now, let $r \geq 2s$ be the smallest element of $S$ greater than $s$. Then, $$\frac{1}{2s} \leq \frac{1}{s}-\frac{1}{r}= \sum_{x \in F} \frac{1}{x}$$for some subset $F$ of $S$. If $r \not \in F$, then $\frac{1}{s}=\sum_{x \in F \cup \{r\}} \frac{1}{x}$, a contradiction, since $s \in S$. Thus, $r \in F \implies \frac{1}{r} \geq \frac{1}{s}-\frac{1}{r}= \sum_{x \in F} \frac{1}{x} \geq \frac{1}{r} \implies \frac{1}{s}-\frac{1}{r}=\frac{1}{r} \implies r=2s$.
By the same argument, we can prove that $S=\{s,2s,2^2s,2^3s,...\}$. If $s>2$, then $\sum_{x \in S} \frac{1}{x}$ converges to a number less than $1$, so we can take a rational number in the interval $(\sum_{x \in S} \frac{1}{x},1)$ ($\mathbb{Q}$ is dense in $\mathbb{R}$) to reach a contradiction.
Thus, $s=1$ or $s=2$. If $s=1$, we can simply ignore $s$ from $S$, so assume WLOG that $s=1$. But then $\frac{1}{3}$ cannot be represented as $\sum_{x \in F} \frac{1}{x}$ where $F$ is a finite subset of $S$, so we are done.
$\blacksquare$
Z K Y
The post below has been deleted. Click to close.
This post has been deleted. Click here to see post.
awesomeming327.
1692 posts
#31
Y by
Let both of the statements be false. We have that for each $r\in (0,1)$ there exists a unique subset $S(r)$ such that \[\sum_{x\in S(r)}\left(\frac{1}{x}\right)=r\]Clearly $S$ must not be finite, since then there is a finite number of possible $S(r)$ but an infinite number of rational numbers, and there cannot be a bijection there. Now, suppose there exist $a,b\in S$ such that $a<b<2a$ then \[\frac{1}{a}-\frac{1}{b}\le \frac{1}{b}\]which means that $b\notin S(\tfrac{1}{a}-\tfrac{1}{b})$ so $\{a\}=S(\tfrac{1}{a})=S(\tfrac{1}{a}-\tfrac{1}{b})\cup \{b\}$, impossible. Now, let any two consecutive terms of $S$ arranged in increasing order be $a>b$. If $a>2b$ then it is easy to see that there is no way to represent $r\in(\tfrac{2}{a},\tfrac{1}{b})$. If we have nothing smaller than $a$ then the maximum we can get is \[\frac{1}{a}+\frac{1}{2a}+\frac{1}{4a}+\dots = \frac{2}{a}\]and if we have something smaller than $a$ then the minimum we can get is $\tfrac{1}{b}$. Therefore, we must have \[S=\{a,2a,4a,8a,\dots\}\]and the maximum sum will be $\tfrac{1}{a}$ so $a=1$. Since we have found $S$, we can very confidently say that there are so many rational numbers that don't have a corresponding subset.
Z K Y
The post below has been deleted. Click to close.
This post has been deleted. Click here to see post.
asdf334
7586 posts
#32
Y by
Amazing!

Assume that neither assertion holds. Then suppose FTSOC that $x,y\in S$ satisfy $x<y<2x$. By considering
\[\frac{1}{x}=\frac{1}{y}+\left(\frac{1}{x}-\frac{1}{y}\right)\]we can actually generate two different subsets of $S$, violating the first condition.

However we also know that if we sum the reciprocals of all elements $x\in S$ satisfying $x>1$ we obtain a value which is at least $1$ (else we violate the second condition). This implies that these elements are exactly the powers of $2$. However by considering base $2$ we find that $\frac{1}{3}$ isn't achieveable so we are done. $\blacksquare$
This post has been edited 1 time. Last edited by asdf334, Mar 11, 2023, 3:49 AM
Z K Y
The post below has been deleted. Click to close.
This post has been deleted. Click here to see post.
sman96
136 posts
#33
Y by
Let, $S = \{x_1, x_2, x_3, \dots\}$ where, $x_1 < x_2 < \dots < x_n < x_{n+1} < \dots$
Now, there are two cases,

$\textbf{Case 1:}$ There exists $k$ s.t. $x_{k+1} < 2x_k$
If the second condition doesn't hold, for all $1 > r \in \mathbb Q$ we will get a finite subset $F_r$ that gives the reciprocal sum $r$.
Now, take $r = \dfrac{x_{k+1} - x_k}{x_kx_{k+1}}$. Then by our condition we get $r < \dfrac1{x_{k+1}}$. So, $F_r$ doesn't contain any $x_n$ with $n \leq k+1$.
Now take, $F = \{x_k\}$ and $G = \{x_{k+1}\} \cup F_r$ and we get our first condition true. $\blacksquare$

$\textbf{Case 2:}$ For all $k$ we have $x_{k+1} \geq 2x_k$
We want to show that the second condition holds.
Notice that, For the second condition it doesn't matter if $1$ is in $S$ or not. So we assume that $1 \notin S$.
Then, $x_1 \geq 2$ and if we get $x_{k+1} > 2x_k$ at some point then, let $c = \dfrac1{2x_k} - \dfrac1{x_{k+1}}$ and we will get that the total sum of reciprocals of $S$ is at most $1-c$.
Then we can just take a $r = 1-c + \epsilon$ for small enough $\epsilon > 0$ and we will be done. And similarly if $x_1 \neq 2$ we get a bound $\frac2{x_1}$ on the total sum.
So now the remaining case is $x_i = 2^i$ for all $i$.
But then we take $r = (0.\overline{01})_2 = (1/3)_{10}$ and if we get a set $F_r$ for this then we will get a finite representation of $r$ in binary which we can easily show that doesn't exist. So we are done. $\blacksquare$
Z K Y
The post below has been deleted. Click to close.
This post has been deleted. Click here to see post.
lrjr24
966 posts
#34
Y by
Assume not. Note that $S$ is infinite because otherwise there is only finitely many sums attainable but there is infinite rationals between $0$ and $1$. Enumerate the elements of $S$ as $s_1 < s_2 < \dots$. Let $p(F) = \sum_{x \in F} \frac{1}{x}$. Then for all rational $r<1$ there is a finite set $F \in S$ such that $p(F) = r$ and for all pairs of finite sets $F,G \in S$, $p(F) \neq p(G)$.

Let's assume that $s_{k+1} < 2s_k$ for some $k$. Then consider the assertion of ($2$) for $\frac{1}{s_k}-\frac{1}{s_{k+1}}$. Note that this value is less than $\frac{1}{s_{k+1}}$, so the finite set $F$ satisfying the condition only contains terms greater than $s_{k+1}$. Let $F$ be the satisfying set. Then $p\left(F \cup \{s_{k+1}\} \right) = \frac{1}{s_k} = p\left(\{ s_k \}\right)$, a contradiction to $(1)$ and so we're done with this case.

Thus $s_{k+1} \ge 2s_k$ for all $k$. Consider a construction for ($2$). Note that $1$ isn't in it as that would make it too big. WLOG $s_1>1$ (if it's $1$ just chop it off). Note that $\sum_{x \in S} \frac{1}{x}$ converges since it's bounded by $\sum_{i=1}^{\infty} \frac{1}{2^i} = 1$. If $s_{k+1} > 2s_k$ for some $k$, $\sum_{x \in S} \frac{1}{x} = \sum_{i=1}^{k} \frac{1}{s_i} + \sum_{i=k+1}^{\infty} \frac{1}{s_i} \le \sum_{i=1}^k \frac{1}{2^i} + \sum_{i=k+1}^{\infty} \frac{1}{2^i+1} < \sum_{i=1}^{\infty} \frac{1}{2^i} = 1$. Thus if we let $r = \sum_{x \in S} \frac{1}{x}$, we won't be able to find a set summing to that (they would all be smaller). Thus $s_{k+1}=2s_k$ and $s_k=2^k$. We finish by taking the assertion for $r=\frac{1}{3}$ and noting that the denominator in any sum would be a power of $2$, which $3$ obviously isn't. This provides a contradiction and concludes the proof.
This post has been edited 1 time. Last edited by lrjr24, Jun 12, 2023, 8:05 PM
Z K Y
The post below has been deleted. Click to close.
This post has been deleted. Click here to see post.
IAmTheHazard
5001 posts
#35 • 1 Y
Y by centslordm
Let $s(T)$ denote the sum of the reciprocals of some set $T$. Suppose that neither of these conditions hold. Clearly $S$ is infinite, and let its elements other than $1$ be $1<a_1<a_2<\cdots$. The key claim is the following.

Claim: We need $a_{i+1} \geq 2a_i$ for all $i$.
Proof: Suppose otherwise. Then there exists some finite $T \subset S$ such that $s(T)=1/a_i-1/a_{i+1}<1/a_{i+1}$, so $a_{i+1} \not \in T$. But then $s(T \cup \{a_{i+1}\})=s(\{a_i\})$: contradiction. $\blacksquare$

This implies that $\sum_{x \in S} 1/s$ converges and is at most $\sum_{i \geq 1} 1/2^i=1$, with equality iff $S=\{2,4,8,\ldots,\}$. But it must be exactly $1$ as well, since we should be able to form numbers arbitrarily close to $1$, so we need equality. But then we can only form dyadic rationals, so this contradicts condition 2. $\blacksquare$
Z K Y
The post below has been deleted. Click to close.
This post has been deleted. Click here to see post.
AdventuringHobbit
164 posts
#36
Y by
Suppose not. Redefine $S$ as a set of reciprocals of whole numbers. Then we can define a function $f(x)$ from rational numbers less than $1$ to finite subsets of $S$ such that $f(x)$ is the unique finite subset whose elements add to $x$. For the rest of the argument I will give, assume WLOG that $1$ is not in $S$ (this does not change the conclusion). Let $s_i$ denote the $i$th largest element in $S$. Observe that $f(s_i)=\{s_i\}$. This means that for $s_i > s_j$, we have $s_j \in f(s_i-s_j)$. Otherwise we could write $s_i=\sum f_(s_i-s_j) \cup \{s_j\}$. A similar argument shows $s_j \notin f(s_i-2s_j)$. We can repeat this. In general, $s_j \in f(s_i-(2n+1)s_j)$ for an integer $n$. Thus, for integers $n$, if $s_i-(2n+1)s_j>0$, then $s_i-(2n+1)s_j \ge s_j$. This means that $\lfloor \frac{s_i}{s_j} \rfloor$ is even for all $i, j$. This implies that if $a \in S$, the next smallest element that could possible be in $S$ is $\frac{a}{2}$. Thus the set $S$ which achieves maximal total sum is the set consisting of fractions which are negative powers of $2$, which has sum exactly $1$. If the set $S$ is not this set, then its elements sum to less than $1$, so we can find a contradiction by finding a rational number $p$ such that $p$ is bigger than the sum of $S$, whence $f(p)$ is undefined. If our set $S$ is the set of negative powers of $2$, then $f(\frac{1}{3})$ is not defined, because of the requirement that the subsets be finite, and $\frac{1}{3}$ in binary is $0.0111\ldots$.
Z K Y
The post below has been deleted. Click to close.
This post has been deleted. Click here to see post.
pi271828
3363 posts
#37 • 1 Y
Y by OronSH
Assume FTSOC there exists an $S$ that doesn't satisfy either assertion. Clearly $S$ must have infinite cardinality. Label the elements of $S$ as $a_1 < a_2 < a_3 \dots$. Notice that for any $x,y \in S$ and $x>y$, there must exist a subset $F$ that has the reciprocals of its elements sum to $\frac{1}{y} - \frac{1}{x}$. Now if $x \notin F$, then when we simply add $x$ to $F$, this new set will have its sum be equal to $\frac{1}{y}$ which is it not possible due to the 1st assumption as we can simply take the set $\{ y \}$. Therefore $x$ must be in $F$, which implies that $\frac{2}{x} \le \frac{1}{y}$, and $x \ge 2y$. Therefore if we have two elements $x > y$, then we must have $x \ge 2y$. From this we easily get $a_{i+1} \ge 2 \cdot a_i$. Now notice that $$\sum_{x \in S} \frac{1}{x} \le \sum_{i=0}^{\infty} \frac{1}{2^i \cdot a_{1}} = \frac{2}{a_{1}}$$Now we have $$\sum_{x \in S} \frac{1}{x} \ge 1$$because if not, for all $F \subset S$ we will have $\sum_{x \in F} \frac{1}{x} < 1$, which contradicts second assumption. Now this simply implies that $a_{1} = 1, 2$ and that $a_{i+1} = 2 \cdot a_{i}$. Now simply take a rational with denominator not a power of $2$. This set cannot achieve such a rational, so we have achieved a contradiction.
This post has been edited 2 times. Last edited by pi271828, Nov 24, 2023, 2:55 AM
Z K Y
The post below has been deleted. Click to close.
This post has been deleted. Click here to see post.
lelouchvigeo
176 posts
#38
Y by
Nice problem. It looks like a difficult problem, but it isn't.
I have solved this in the same way as (4) :D
Therefore not uploading
This post has been edited 1 time. Last edited by lelouchvigeo, Jan 11, 2024, 12:20 PM
Z K Y
The post below has been deleted. Click to close.
This post has been deleted. Click here to see post.
dkedu
180 posts
#39
Y by
AFTSOC that both statements are true.

Claim: $\frac{1}{s_1} = \sum_{i\ge2}\frac{1}{s_i}$.

If $\frac{1}{s_1} > \sum_{i\ge2}\frac{1}{s_i}$, then there exists a rational number $r$ between $1$ and $\sum_{i\ge1}\frac{1}{s_i}$ since $\sum_{i\ge1}\frac{1}{s_i} < \frac{2}{s_1} \le 1$ and the rationals are dense in the reals.

If $\frac{1}{s_1} < \sum_{i\ge2}\frac{1}{s_i}$, then there must exist a $k$ such that $\frac{1}{s_1} < \sum_{i=2}^k \frac{1}{s_i}$ and $\frac{1}{s_1} > \sum_{i=2}^{k-1} \frac{1}{s_i}$. Now by assumption, there exists a set $A$ such that $\sum_{a\in A} \frac{1}{s_i} = \left(\sum_{i=2}^k \frac{1}{s_i}\right) - \frac{1}{s_1} < \frac{1}{s_k}$. So all elements of $A$ must be less than $k$, so we can just take $\{1\} \cup A$ and $\{2, \dots, k\}$ to get a contradiction, so we have proven the claim.

Now note that $s_1= 2$ otherwise $r = 1 - \epsilon$ gives a contradiction. Now this argument clearly works for $s_2$ as well, by only considering numbers under $\frac{1}{2}$ so we get that $s_i = \frac{1}{2^i}$. Now taking $r = \frac{1}{3}$ gives a contradiction so we are done.
Z K Y
The post below has been deleted. Click to close.
This post has been deleted. Click here to see post.
MathLuis
1475 posts
#40
Y by
The problem is trivial if $S$ is finite, so suppose it's infinite, FTSOC assume that neither of the claims was true, then trivially all elements of $S$ are distinct from each other so we let $S= \{a_1, a_2, \cdots \}$ where we have them ordered in increasing order, meaning that $a_i<a_{i+1}$ and also note that $a_1 \ge 2$ since if a set had $1$ and satisfied the condition then so will $S-{1}$, so it doesn't really matter.
First we take two elements $a_k>a_j$ (so of course $k>j$), then consider $\sum_{x \in S_{k,j}} \frac{1}{x}=\frac{1}{a_j}-\frac{1}{a_k}$, suppose FTSOC that $2a_j>a_k$, then it's clear that in $S_{k,j}$ a subset of $S$ we can't have $a_k$ on it for size reasons, so now note that we have $\sum_{x \in S_{k,j} \cup (a_k)} \frac{1}{x}=\frac{1}{a_j}$ which contradicts the opposite of (1), therefore we have $a_k \ge 2a_j$, this in general implies that $a_{i+1} \ge 2a_i$ for any $i \in \mathbb Z_{>0}$.
Now suppose FTSOC that we have some $i$ for which $a_{i+1}>2a_i$, then consider a rational number $r$ strictly between $\frac{2}{a_{i+1}}$ and $\frac{1}{a_i}$, then let $S_1$ be the finite subset such that $\sum_{x \in S_1} \frac{1}{x}=r$, notice that for size reasons we can't have any $a_1, \cdots a_i$ on $S_1$ therefore $\sum_{x \in S_1} < \frac{1}{a_{i+1}} \left(1+\frac{1}{2}+\cdots \right)=\frac{2}{a_{i+1}}$ which contradicts the existence of $r$, therefore $a_{i+1}=2a_i$ must be true for all $i \in \mathbb Z_{>0}$.
Therefore we have that $S$ is of the form $(\ell, 2\ell, 4\ell, \cdots )$ but now it's impossible to create $\frac{2}{\ell}$ for all $\ell \ge 3$ meaning that either $\ell=1$ or $2$, but since we can get rid of $1$ we can just consider $\ell=2$ in which case we have $S$ of the form $(2, 4, 8 \cdots)$ in which from $v_2$ and base $2$ we can check that it happens to be impossible to make $\frac{1}{3}$, thus we are done :cool:.
Z K Y
The post below has been deleted. Click to close.
This post has been deleted. Click here to see post.
Pitchu-25
53 posts
#41
Y by
Suppose ftsoc that a set $S$ doesn't satisfy any of the two conditions. Clearly, $S$ must be infinite. For a rational number $r\ge 0$, call its decomposition $F\subset S$ if $\sum_{x\in F}\frac{1}{x}=r$. Any rational number $r<1$ has a unique decomposition, and no rational number has more than one decomposition. We start with a fairly simple observation :

Claim : For any element $s\in S$ and any rational number $r\in \left[\frac{1}{s}; \frac{2}{s}\right[$, $r$ has a unique decomposition, which contains $s$. Moreover, if any rational number $r\in \left[\frac{2}{s}; \frac{3}{s}\right[$ has a decomposition, it does not contain $s$.
Proof : For the first part : note that $r-\frac{1}{s}\in \left[0;\frac{1}{s}\right[$. Therefore $r-\frac{1}{s}$ has a unique decomposition, and it cannot contain $s$. It follows that $r$ has a decomposition which contains $s$.
Now, suppose some $r\in \left[\frac{2}{s}; \frac{3}{s}\right[$ has a decomposition which contains $s$. Then, $r-\frac{1}{s}\in \left[\frac{1}{s}; \frac{2}{s}\right[$ has a decomposition which doesn't contain $s$, which is a contradiction. $\square$

This leads to the following claim :

Claim : Suppose $S=\{a_1,a_2,\ldots\}$ where $a_1<a_2<\ldots$. Then, for each positive integers $k$ and $n$, we have $a_{k+1}\ge 2a_k$ and $\frac{1}{a_k}>\sum_{i=1}^{n}\frac{1}{a_{k+i}}$.
Proof : Note that $r=\frac{1}{a_{k}}+\frac{1}{a_{k+1}}>\frac{2}{a_{k+1}}$ is a rational number which contains $a_{k+1}$ in its decomposition. Therefore, it must be the case that $r\ge \frac{3}{a_{k+1}}$ by our claim, which gives $a_{k+1}\ge 2a_k$. Now, suppose that some $k$ and $n$ are such that $\frac{1}{a_k}\le \sum_{i=1}^{n}\frac{1}{a_{k+i}}$, and take the minimal such $n$. We then have $\sum_{i=1}^{n}\frac{1}{a_{k+i}}\in \left[\frac{1}{a_k}; \frac{2}{a_k}\right[$, which contradicts our claim since its decomposition must contain $a_k$. $\square$

Let us fix some $k\in \mathbb N$. The sequence $x_n=\sum_{i=1}^{n}\frac{1}{a_{k+i}}$ is increasing and bounded above, so it must have a limit $l=l(k)$, which must not be larger than $a_k$ by our claim. If $l<\frac{1}{a_k}$, then any rational number $q\in \left]l;\frac{1}{a_k}\right[$ does not have a decomposition. This is a contradiction, since every number in this interval is smaller than $1$.

Therefore, we must have $l(k)=\frac{1}{a_k}$. Since $a_{k+i}\ge 2^ia_k$, it must be the case that $a_{k+i}=2^ia_k$ for every $i$ and $k$. If not, then clearly $l(k)<\frac{1}{a_k}$ by an easy geometric series argument. In particular, we have $a_k=2^{k-1}a_1$ for every $k$. This cannot hold, since $\frac{1}{3a_1}$ would not have a decomposition. $\blacksquare$
This post has been edited 1 time. Last edited by Pitchu-25, Dec 30, 2024, 1:44 PM
Z K Y
The post below has been deleted. Click to close.
This post has been deleted. Click here to see post.
maromex
148 posts
#42
Y by
Assume the negative of (1) and the negative of (2) for the sake of contradiction, and name these hypotheses (n1) and (n2) respectively. We get that for every rational $r<1$, there is unique $F$ such that $\sum\limits_{x\in F} \frac{1}{x} = r$. In particular, (n2) tells us there is at least one set $F$ satisfying the condition and (n1) tells us there is at most one set $F$ satisfying the condition. This means we can define $f(r) = F$ as described here. The domain of $f$ is all positive rational numbers less than 1.

Let $a_0, a_1, a_2, \ldots$ be the elements of $S$ that are not equal to 1 in increasing order
Claim (3): $a_{i+1} \ge 2a_i$ for all nonnegative integers $i$
Proof: By contradiction; assume $a_{i+1} < 2a_i$. Then, $\frac{1}{a_i} - \frac{1}{a_{i+1}} < \frac{1}{a_i} - \frac{1}{2a_i} = \frac{1}{2a_i} < \frac{1}{a_{i+1}}$. Thus, $a_{i+1}$ cannot be in the set $f(\frac{1}{a_i} - \frac{1}{a_{i+1}})$, because if so, it would cause $\sum\limits_{x \in f(\frac{1}{a_i} - \frac{1}{a_{i+1}})} \frac{1}{x} > \frac{1}{a_i} - \frac{1}{a_{i+1}}$. We may let $G = f(\frac{1}{a_i} - \frac{1}{a_{i+1}}) \cup \{ a_{i+1} \}$, and $G$ satisfies $\sum\limits_{x \in G} \frac{1}{x} = \frac{1}{a_i} = \sum\limits_{x \in \{ a_i \}} \frac{1}{x}$, contradicting (n1).

Claim (4): $a_{i+1} \le 2a_i$ for all nonnegative integers $i$
Proof: By contradiction; assume $a_{i+1} > 2a_i$. This implies $a_{i+1} \ge 2a_i + 1$. Repeated applications of (3) replacing $i$ with $j+1, j+2, \ldots j+k$ shows that $a_{j+k+1} \ge 2^ka_{j+1}$ for all nonnegative integers $j, k$. Thus, $\sum\limits_{k=0}^{|S|} \frac{1}{a_{i+k+1}} \le \sum\limits_{k=0}^{|S|} \frac{1}{2^ka_{i+1}} \le \frac{2}{a_{i+1}} \le \frac{2}{2a_i + 1} < \frac{1}{a_i}$. Let $r_i$ be a rational number such that $\frac{2}{2a_i + 1} < r_i < \frac{1}{a_i}$. The set $f(r_i)$ cannot include $a_i$ or any term before it, because if so, they would cause $\sum\limits_{x \in f(r_i)} \frac{1}{x} > r_i$. However, we have shown that, if $f(r_i)$ includes only terms after $a_i$, then $\sum\limits_{x \in f(r_i)} \frac{1}{x} \le \frac{2}{2a_i+1} < r_i$. Thus, the set $f(r_i)$ does not exist, contradicting (n2).

Combining (3) and (4) will give us Claim (5): $a_{i+1} = 2a_i$ for all nonnegative integers $i$
Let $p \neq 2$ be a prime that does not divide $a_0$.

Repeated application of (5) with $i=0, 1, \ldots k-1$ gives $a_k = 2^k a_0$ for all nonnegative integers $k$. Because $p$ is not 2 and does not divide $a_0$, it does not divide $2^k a_0$ either. Therefore, $p$ does not divide $a_k$ for all nonnegative integers $k$. We must have $\sum\limits_{x \in f(\frac{1}{p})} \frac{1}{x} = \frac{1}{p}$. After cross-multiplying this sum, we see that $p$ divides $\prod\limits_{x \in f(\frac{1}{p})} x$. This implies that there exists an $x$ such that $p$ divides $x$. This contradicts our result that $p$ does not divide $a_k$ for all nonnegative integers $k$, finishing our proof.
This post has been edited 1 time. Last edited by maromex, Apr 1, 2025, 1:57 AM
Reason: Forgot the not equal to 1 condition
Z K Y
N Quick Reply
G
H
=
a